A study found that the expected annual income in a certain area is $17,255. Which of the following statistical measurements most likely led to this conclusion? Mean, range, median or mode?

Answers

Answer 1

Answer:

Mean

Step-by-step explanation:

mean is the average of numbers put together and divided by the total amount of numbers, when finding the average annual income using the mean would be most effective


Related Questions

1.Solve by factorization method: x+1/x=11 1/11 2.Comment on the nature of roots for 4x^2-5=2(〖x+1)〗^2-7 plz, help...

Answers

Answer:

The equation

[tex]4\,x^2-5=2\,(x+1)^2-7[/tex]

can be solved by first expanding all indicated operations, and later when the constant terms disappear, by factoring out 2x , leaving the equation as a product of two factors equal zero, from which it is easy to extract the roots. See below.

Step-by-step explanation:

When solving for x in the following expression, and using factoring to apply at the end the zero product theorem:

[tex]4\,x^2-5=2\,(x+1)^2-7\\4\,x^2-5=2\,(x^2+2x+1)-7\\4\,x^2-5=2\,x^2+4\,x+2-7\\4\,x^2-5=2\.x^2+4\,x-5\\4\,x^2=2\,x^2+4\,x\\4\,x^2-2\,x^2-4\,x=0\\2\,x^2-4\,x=0\\2\,x\,(x-2)=0[/tex]

We observe that for the last product, to get a zero, x has to be zero (making the first factor zero), or x has to be "2" making the binomial factor zero.

The cost of milk is modeled by a linear equation where four quarts (one gallon) costs $3.09 while two quarts
(half-gallon) costs $1.65. Write the linear equation that expresses the price in terms of quarts. How much would
an eight-quart container of milk cost?

Answers

Answer:

linear equation to express the price is:

y=0.72x+0.21

An eight quarts will cost :  $5.97

Step-by-step explanation:

linear equation represent y=mx+b

let x=quarts ( x=4, x=2)

y= price (3.09 and y=1.65 )

two points (4,3.09) and (2,1.65)

need to find the slope m:

y2-y1/x2-x1

(1.65-3.09)/(2-4) ⇒ m=0.72

y=0.72x+b  find b at point (2,1.65)

1.65=0.72(2) +b  ⇒ b=0.21

y=0.72x +0.21

check : point (4,3.09)

y=0.72(4) +0.21

y=3.09  ( correct)

An eight quarts will cost :

y=0.72(8)+0.21

y=5.97 dollars

James conducted an experiment with 4 possible outcomes. He determined that the experimental probability of event A happening is 10 out of 50. The theoretical probability of event A happening is 1 out of 4. Which action is most likely to cause the experimental probability and theoretical probabilities for each event in the experiment to become closer? removing the last 10 trials from the experimental data completing the experiment many more times and combining the results to the trials already done including a fifth possible outcome performing the experiment again, stopping immediately after each event occurs once

Answers

Answer:

Completing the experiment a few more times and combining the results to the trails already done.

Answer:

Completing the experiment a few more times and combining the results to the trails already done.

Step-by-step explanation:

A laundry basket contains 18 blue socks and 24 black socks. What is the probability of randomly picking 2 black socks, with replacement, from the basket?

Answers

Answer:

144/441

Step-by-step explanation:

There are 18+24=42 total socks

There are 24 black socks

So the probability is (24/42)*(24/42)=12/21 * 12/21 = 144/441

Answer:

189

Step-by-step explanation:

1. In a right triangle, the lengths of the legs are a and b. Find the length of a hypotenuse, if: a=1, b=1; 2. In a right triangle, the length of a hypotenuse is c and the length of one leg is a. Find the length of the other leg, if: c=5, a=3;

Answers

Answer:

1. [tex]c = \sqrt{2}[/tex].

2. b = 4.

Step-by-step explanation:

To solve these two questions, keep the Pythagorean Theorem in mind: [tex]a^2 + b^2 = c^2[/tex]. Also remember that measurements cannot be negative, so we will disregard the negative answers.

1. a = 1, and b = 1. c = ?

[tex]1^2 + 1^2 = c^2[/tex]

[tex]1 + 1 = c^2\\[/tex]

[tex]c^2 = 1 + 1\\c^2 = 2\\\sqrt{c^2} = \sqrt{2}\\c = \sqrt{2}[/tex]

2. a = 3, c = 5. b = ?

[tex]3^2 + b^2 = 5^2\\9 + b^2 = 25\\b^2 = 16\\\sqrt{b^2} = \sqrt{16}\\b = 4[/tex]

Hope this helps!

|3x–1|=8 please help!!!!!

Answers

Answer: -3

Add 1 to both sides

[tex]3x-1+1=8+1[/tex]

[tex]3x=9[/tex]

Divide both sides by 3

[tex]3x/3=9/3\\x=3[/tex]

Please solve (will make brainiest)

Answers

Answer:

1a) 1/64

1b) 1/169

1c) 1/9

Step-by-step explanation:

You have to apply Indices Law :

[tex] {a}^{ - n} = \frac{1}{ {a}^{n} } [/tex]

Question A,

[tex] {4}^{ - 3} = \frac{1}{ {4}^{3} } = \frac{1}{64} [/tex]

Question B,

[tex] {13}^{ - 2} = \frac{1}{ {13}^{2} } = \frac{1}{169} [/tex]

Question C,

[tex] {( - 3)}^{ - 2} = {( - \frac{1}{3}) }^{2} = \frac{1}{9} [/tex]

1: The best statement for reason 6 of this proof is -∠A ≅ ∠C
-∠B ≅ ∠D
-∠B and ∠D are supplements
-∠B ≅ ∠B

2.The best reason for statements 3.5. and 7 in this proof is
- Alternate interior angles are congruent.
-Corresponding angles are congruent.
-Alternate exterior angles are congruent.
-Interior angles on the same sides of a transversal are supplements.

3. The best statement for reason 8 of this proof is
-∠B ≅ ∠B -∠A and ∠C are supplements.
-∠B ≅ ∠D
-∠A ≅ ∠C

Answers

Answer:

1) -∠B ≅ ∠D

2) -Interior angles on the same side of a transversal are supplementary

3) -∠A ≅ ∠C

Step-by-step explanation:

1) Given that ∠A and ∠B are supplements and ∠A and ∠D are supplements, we have; ∠B ≅ ∠D

2)  Given that ABCD is a parallelogram, therefore ∠A and ∠B,  ∠A and ∠D and ∠B and ∠C  are interior angles on the same side of a transversal and are therefore supplementary

3) Given that ∠A and ∠B and ∠B and ∠C are supplementary, therefore, ∠A ≅ ∠C.

HELP ASAP PLEASE!!!!!!!!!!!!!!!!!

Answers

Answer:

A

C

D

Step-by-step explanation:

√54 or√9 *√6 or √27 *√4

are equal to the answer.

You can do that by doing the square of outer number which is 3 which equals to 9 when squared and multiplying that with the number inside the square root.

Find the measure of each side indicated. Round to the nearest tenth.

A) 19.8
C) 24.9
B) 27.2
D) 25.3

Answers

Answer:

D. 25.3

Step-by-step explanation:

tan∅ = opposite over adjacent

Step 1: Write equation

tan66.5° = x/11

Step 2: Multiply both sides by 11

11tan66.5° = x

Step 3: Evaluate

x = 25.2983

x ≈ 25.3

Answer:

[tex]\huge\boxed{x = 25.3}[/tex]

Step-by-step explanation:

Tan θ = opposite / adjacent

Where θ = 66.5 , opposite = x and adjacent = 11

Tan 66.5 = x / 11

2.3 * 11 = x

25.3 = x

OR

x = 25.3

A shop sells DVDs and CDs.

DVDs are sold at one price.
CDs are sold at a different price.

2 DVDs and 1 CD cost £35
2 DVDs and 2 CDs cost £45

Martin has £50 Does he have enough to buy 1 DVD and 3 CDs?

Answers

Answer:

Step-by-step explanation:

Lets Price of a DVD is fixed i.e. 15

and One CD price is 5 (Not fixed)

In First situation

2 DVDs and 1 CD cost = 35 as given

2 x 15 + 5 = 35

Lets one CD price is 7.5

In Second situation

2 x 15 + 2 x 7.5 = 45

Its mean CD price may be between 5 to 7.5

In asked scenario, Martin has 50

1 DVD and 3 CDs?

1 x 15 + 3 x 7.5 = 37.5

37.5 is lesser than 50

Hence Martin has enough to buy 1 DVD and 3 CDs.


31. Each day, Talisa exercises by first
stretching and then swimming
some laps, as shown in the table.
Make a scatter plot of the total
time she exercises as a function
of the number of laps she swims.
Draw a trend line.

Answers

Answer:

Step-by-step explanation:

Given the following :

Laps - - - - - - - - 5 - - - 6 - - - 7 - - - 8 - - - 9

Total time - - - 25 - - 28 - - 29 - - 30 - - 32

Using online graphing tool:

The y - axis named dependent variable represents the total time taken.

The x-axis, represents the number of laps.

The equation of the trend line attached to the plot is in the form :

y = mx + c

y = 1.6x + 17.6

Where y = total time taken

x = number of laps

m = 1.6 = gradient of the line (change in y / change in x)

C = 17.6 = intercept (whee the trndline intersects the y-axis).

. Find two polynomial expressions whose quotient, when simplified, is 1/x . Use that division problem to determine whether polynomials are closed under division.

Answers

Answer:

The two polynomials are:

(x + 1) and (x² + x)

Step-by-step explanation:

A polynomial is simply an expression which consists of variables & coefficients involving only the operations of addition, subtraction, multiplication, and non - negative integer exponents of variables.

Now, 1 and x are both polynomials. Thus; 1/x is already a quotient of a polynomial.

Now, to get two polynomial expressions whose quotient, when simplified, is 1/x, we will just multiply the numerator and denominator by the same polynomial to get more quotients.

So,

Let's multiply both numerator and denominator by (x + 1) to get;

(x + 1)/(x(x + 1))

This gives; (x + 1)/(x² + x)

Now, 1 and x are both polynomials but the expression "1/x" is not a polynomial but a quotient and thus polynomials are not closed under division.

I NEED YOUR HELP PLS

Answers

Answer:

For question 1 you can try dividing each of the value

For instance, you can divide 9 by 25 and see if you get a nice number

e.g. 1/8=0.125, numbers like these

For the second question, you can find the fraction by dividing 1000 starting with the decimal points

e.g 0.650, you would be plotting 650/1000 and you would simplify the fraction to the lowest value any value above the decimal point you can multiply by the denominator and add the nominator value to get your final answer.

Step-by-step explanation:

Answer:

Write the denominator in its prime factors. If the prime factorization of the denominator of a fraction has only factors of 2 and factors of 5, the decimal expression terminates.  If there is any prime factor in the denominator other than 2 or 5, then the decimal expression repeats.

example: 9/25

25 = 5*5, so it will be terminating

example: 7/12

12 = 3*2*2, which contains a 3, so it will be repeating.

LCM of x<sup>2</sup>+5x+6 and x<sup>2</sup>-x-6 is ………………………





Answers

Answer:

[tex] (x^2 - 9)(x + 2) [/tex]

Step-by-step explanation:

Given:

[tex] x^2 + 5x + 6 [/tex]

[tex] x^2 - x - 6 [/tex]

Required:

LCM of the polynomials

SOLUTION:

Step 1: Factorise each polynomial

[tex] x^2 + 5x + 6 [/tex]

[tex] x^2 + 3x + 2x + 6 [/tex]

[tex] (x^2 + 3x) + (2x + 6) [/tex]

[tex] x(x + 3) + 2(x + 3) [/tex]

[tex] (x + 2)(x + 3) [/tex]

[tex] x^2 - x - 6 [/tex]

[tex] x^2 - 3x +2x - 6 [/tex]

[tex] x(x - 3) + 2(x - 3) [/tex]

[tex] (x + 2)(x - 3) [/tex]

Step 2: find the product of each factor that is common in both polynomials.

We have the following,

[tex] x^2 + 5x + 6 = (x + 2)(x + 3) [/tex]

[tex] x^2 - x - 6 = (x + 2)(x - 3) [/tex]

The common factors would be: =>

[tex] (x + 2) [/tex] (this is common in both polynomials, so we would take just one of them as a factor.

[tex] (x + 3) [/tex] and,

[tex] (x - 3) [/tex]

Their product = [tex] (x - 3)(x + 3)(x +2) = (x^2 - 9)(x + 2) [/tex]

There are 9 classes of 25 students each, 4 teachers, and two times as many chaperones as teachers.
Each bus holds a total of 45 people.
What is the least number of buses needed for the field trip?

Answers

5 buses is the answer pls mark me brainliest

Least number of bus require for trip = 5 buses

What is Unitary method?

It is a method where we find the value of a single unit from the value of multiple units and the value of multiple units from the value of a single unit.

Steps to Use Unitary Method

First, let us make a note of the information we have. There are 5 ice-creams. 5 ice-creams cost $125.

Step 1: Let’s find the cost of 1 ice cream. In order to do that, divide the total cost of ice-creams by the total number of ice-creams. The cost of 1 ice-cream = Total cost of ice-creams/Total number of ice-creams = 125/5 = 25. Therefore, the cost of 1 ice cream is $25.

Step 2: To find the cost of 3 ice-creams, multiply the cost of 1 ice cream by the number of ice-creams. The cost of 3 ice-creams is cost of 1 ice-cream × number of ice-creams = 25 × 3 = $75. Finally, we have the cost of 3 ice-creams i.e. $75.

Given:

Total number of classes = 9

Number of student in each class = 25

Number of teacher = 4

Number of chaperones = Double of teacher

Bus hold = 45 people

Now,

Total number of student = 9 × 25

                                         = 225

Number of chaperones = 4 × 2

                                         = 8

Total people = 225 + 8 + 4

                     = 237

Least number of bus require for trip = Total people / Bus hold

= 237 / 45

= 5.266

Learn more about unitary method here:

https://brainly.com/question/22056199

#SPJ2

Find the missing probability. P(A)=1120,P(B|A)=1320,P(A∩B)=?

Answers

Answer: 143/400

Explanation:

Assuming you meant to say

P(A) = 11/20

P(B|A) = 13/20

then,

P(A∩B) = P(A)*P(B|A)

P(A∩B) = (11/20)*(13/20)

P(A∩B) = (11*13)/(20*20)

P(A∩B) = 143/400

THE ANSWER IS 143/400 !

I need help on both answers. They’re different from my other problems so I’m kinda confused

Answers

i think it would be b but
i’m not sure about the other one

1)Sheyna drive to the lake and back. It took two hours less time to get there than it did to get back. The average speed on the trip there was 60 mph. The average speed on the way back was 36 mph. How many hours did the trip there take?

Answers

Answer:

8 hours

Step-by-step explanation:

Given:

Sheyna drives to the lake with average speed of 60 mph and

[tex]v_1 = 60\ mph[/tex]

Sheyna drives back from the lake with average speed of 36 mph

[tex]v_2 = 36\ mph[/tex]

It took 2 hours less time to get there than it did to get back.

Let [tex]t_1[/tex] be the time taken to drive to lake.

Let [tex]t_2[/tex] be the time taken to drive back from lake.

[tex]t_2-t_1 = 2[/tex] hrs ..... (1)

To find:

Total time taken = ?

[tex]t_1+t_2 = ?[/tex]

Solution:

Let D be the distance to lake.

Formula for time is given as:

[tex]Time =\dfrac{Distance}{Speed }[/tex]

[tex]t_1 = \dfrac{D}{60}\ hrs[/tex]

[tex]t_2 = \dfrac{D}{36}\ hrs[/tex]

Putting in equation (1):

[tex]\dfrac{D}{36}-\dfrac{D}{60} = 2\\\Rightarrow \dfrac{5D-3D}{180} = 2\\\Rightarrow \dfrac{2D}{180} = 2\\\Rightarrow D = 180\ miles[/tex]

So,

[tex]t_1 = \dfrac{180}{60}\ hrs = 3 \ hrs[/tex]

[tex]t_2 = \dfrac{180}{36}\ hrs = 5\ hrs[/tex]

So, the answer is:

[tex]t_1+t_2 = \bold{8\ hrs}[/tex]

20 squared (+5) divided by 100

Answers

The answer is 4.05

Step-by-step explanation:

20^2 is 20•20 which is 400 || +5=405 || /100=4.05

Can someone please tell me how to solve this problem??!! I literally have to go back in math if I don’t pass this HELP!!

Answers

Answer:

          D.   270° < φ < 360°

Step-by-step explanation:

Imagine coordinate system

I quarter is where x>0 and y>0  {right top} and it is (0°,90°)

II quarter is where x<0 and y>0  {left top} and it is (90°,180°)

III quarter is where x<0 and y<0  {left bottom} and it is (180°,270°)

IV quarter is where x>0 and y<0  {right bottom} and it is (270°,360°)

Now, we have an angle wich vertex is point (0,0) and one of its sides is X-axis and the second lay at one of the quarters.

For the trig functons of an angle created by this second side always are true:

In first quarter all functions are >0

in second one only sine

in third one: tangent and cotangent

and in fourth one: cosine

{You can check this by selecting any point on the second side of angle and put it's coordinates to formulas of these functions:

[tex]\sin \phi=\dfrac y{\sqrt{x^2+y^2}}\,,\quad \cos \phi=\dfrac x{\sqrt{x^2+y^2}}\,,\quad \tan\phi=\dfrac yx\,,\quad \cot\phi=\dfrac xy[/tex]  }

So:

sinφ<0  ⇒ III or IV quarter

tanφ<0  ⇒ I or IV quarter

IV quarter  ⇒  φ ∈ (270°, 360°)

Type the correct answer in the box. Use numerals instead of words. If necessary, use / for the fraction bar. Stacy goes to the county fair with her friends. The total cost of ride tickets is given by the equation c = 3.5t, where c is the total cost of tickets and t is the number of tickets. If Stacy bought 15 tickets, she would spend $

Answers

Answer:

$52.2

Step-by-step explanation:

Given her total cost of ride tickets modeled by the equation c = 3.5t where c is the total cost of tickets and t is the number of tickets, If Stacy bought 15 tickets, to know the amount she would spend on 15 tickets, we will substitute t = 15 into the modeled equation as shown;

[tex]c = 3.5t\\when t = 15\\\\c = 3.5(15)\\\\c = \frac{35}{10} * 15\\ \\c = \frac{5*7}{5*2} * 15\\\\[/tex]

[tex]c = \frac{7}{2} * 15\\ \\c = \frac{105}{2}\\ \\c = \ 52.2[/tex]

Hence Stacy would spend $52.2 on 15 tickets

Answer:

I hope this helps!

Step-by-step explanation:

What is 12.5% of 72

Answers

Answer:

[tex]\boxed{9}[/tex]

Step-by-step explanation:

[tex]\sf of \ refers \ to \ multiplication.[/tex]

[tex]12.5\% \times 72[/tex]

[tex]\frac{12.5}{100} \times 72[/tex]

[tex]\sf Multiply.[/tex]

[tex]\frac{900}{100} =9[/tex]

Can you please Solve for x

Answers

x - 3 = 27

add three to both sides

then x= 30

●✴︎✴︎✴︎✴︎✴︎✴︎✴︎✴︎❀✴︎✴︎✴︎✴︎✴︎✴︎✴︎✴︎✴︎●

         Hi my lil bunny!

❧⎯⎯⎯⎯⎯⎯⎯⎯⎯⎯⎯⎯⎯⎯⎯⎯⎯⎯⎯⎯⎯⎯⎯⎯⎯⎯⎯⎯⎯⎯⎯⎯⎯⎯⎯⎯⎯⎯☙

Let's solve your equation step-by-step.

[tex]x-3=27[/tex]

Step 1: Add 3 to both sides.

[tex]x -3 + 3 = 27 +3[/tex]

[tex]x = 30[/tex]

So the answer is : [tex]x = 30[/tex]

❧⎯⎯⎯⎯⎯⎯⎯⎯⎯⎯⎯⎯⎯⎯⎯⎯⎯⎯⎯⎯⎯⎯⎯⎯⎯⎯⎯⎯⎯⎯⎯⎯⎯⎯⎯⎯⎯⎯☙

●✴︎✴︎✴︎✴︎✴︎✴︎✴︎✴︎❀✴︎✴︎✴︎✴︎✴︎✴︎✴︎✴︎✴︎●

Hope this helped you.

Could you maybe give brainliest..?

❀*May*❀

Remember, a percent is a fractional part
of 100. In a bag of candy, 15 of the 50
pieces are red. What percentage of the
candy is red?
mex
B 50%.
C 3006
D 659​

Answers

Answer:

Step-by-step explanation:

B

Answer:

The answer would be 30% (although I don't see that as an answer).

Step-by-step explanation:

This is because when you multiply the denominator times a number that makes the denominator 100 and multiply that same number by the numerator you get the percentage of the sample you are looking at on the numerator.

15/50 = (15*2)/(50*2) = 30/100 = 30%

At the Olympic games, many events have several rounds of competition. One of these events is the men's 100 100100-meter backstroke. The upper dot plot shows the times (in seconds) of the top 8 88 finishers in the final round of the 2012 20122012 Olympics. The lower dot plot shows the times of the same 8 88 swimmers, but in the semifinal round. Which pieces of information can be gathered from these dot plots? (Remember that lower swim times are faster.) Choose all answers that apply: Choose all answers that apply:

Answers

Answer:

The center of the semifinal round distribution is greater than the center of final round distribution.

The variability in the semifinal round distribution is less than variability in the final round distribution.

Step-by-step explanation:

The mean value of each distribution set is not calculates as the center of semifinal round distribution is greater than the final round distribution. MAD Mean Absolute Deviation is calculated from the dotted graph plot, the distribution of semifinal round is less spread out than the final round distribution.

Answer:

correct answer is None of the above i understood nothing the other person was trying to say...

Step-by-step explanation:

mark me brainliest please...

I answered all my work correctly but I don’t understand this one.

Answers

Take your x values in each coordinate and subtract 2, and take your y values and subtract 1.

Q (0-2), (-1-1)
= (-2,-2)

D (-2-2), (2-1)
= (-4, 1)

V (2-2), (4-1)
= (0,3)

J (3-2), (0-1)
= (1,-1)

You can also draw it on a graph and then translate all coordinates 2 units left and 1 down to see the end results.

Complete the following two-way frequency table.

Answers

Answer:

Step-by-step explanation:

Number of candies with Forest = 12

Candies containing coconut and chocolate both = Number common in coconut and the chocolate = 3

Candies which do not contain coconut but contain the chocolate = 6

Candies which contain the coconut but do not contain the chocolate = 1

Candies which neither contain the chocolate nor coconut = 2

From the given Venn diagram,

                                                Contain coconut         Do not contain coconut

Contain chocolate                              3                                       6

Do not contain chocolate                 1                                        2

What is the rate of change and initial value for the linear relation that includes the points shown in the table?
ху
1 | 20
3 | 10
5 | 0
7 | -10
A. Initial value: 20, rate of change: 10
B. Initial value: 30, rate of change: 10
C. Initial value: 25, rate of change: -5
D. Initial value: 20, rate of change: -10

Answers

Answer:

C, at 0/25, 1/20, 2/15, 3/10,...

Answer:

C

Step-by-step explanation:

Greyson completes a dive from a
cliff 75-feet above a river. It takes
him only 1.5 seconds to hit the
water and then another 0.5
second to descend 10 feet into the river

what’s the x axis and y axis?

Answers

Answer: y: height, x: time.

Step-by-step explanation:

The data we have is:

The initial position of Greyson is 75ft above the river.

He needs 1.5 seconds to hit the water, and other 0.5s tho reach the bottom of the river.

Then we have a relationship of height vs time.

The y axis will represent the heigth of Greyson, and the x-axis will represent the time, such that at the time x = 0 seconds, we have y = 75ft

Other Questions
A balloon has an initial volume of 2.954 L containing 5.50 moles of helium. More helium is added so that the balloon expands to 4.325 L. How much helium (moles) has been added if the temperature and pressure stay constant during this process. 10 points :) Graph this for me :P 1.A big debate is raging in both the religious and political arenas regarding same sex marriages. Do you think that if people choose to be committed together, they are entitled to be recognized as a married couple? Why or why not? Present reasons for your answer. 2. Why do you think preventing spousal and domestic abuse should be a concern of the government? Do you think it should? Back up your answers with evidence from the reading. 3.Do you think couples should be entitled to the same legal benefits as those who have a marriage license? Why or why not? 4.Leona Helmsley left her millions to her nine-year-old dog. She was legally sane at the timeand ran her businesses and corporations until the day she died. Do you think she had the legal right to do what she wanted with her money, or was the judge in the case within his rights to change Leonas will? Why or why not? 5. Explain the importance of a will, a power of attorney document, and a guardianship agreement in the case of a cohabitating coupleparents of two childrenwho are killed in an auto accident. PLEASE HELP ASAP will mark the brainiest paragraph if possible A fish jumps out of the water at a speed of 12 feet per second. The height y (in feet) of the fish above the surface of the water is represented by the equation y=-16x^2+12x, where x is the time (in seconds) since the jump began. The fish reaches its highest point above the surface of the water after 0.375 seconds. How far above the surface is the fish at this time? A cabinet door has a perimeter of 76 inches. Its area is 357 square inches. What are the dimensions of the door? Assume that strike Counter has already been declared to be a "pointer to int". Assume further that strike Counter has been initialized -- its value is the address of some int variable. Write a statement that adds 22 to the value of the variable that strikeCounter is pointing to. HELP ASAP WILL ,ARK BRAINLIEST!!! Does the study of geography only have to do with knowing the physical features of the earth and the atmosphere? Why or why not? Write a letter to your pen friend telling him or her about the effect of corona virus ravaging your country at present Which kingdom of organisms is being described here?1. Cannot move2. Makes it's own food3. Made of more than one cell4. Cells have a nucleus On January 1, 2017, Ann Price loaned $157773 to Joe Kiger. A zero-interest-bearing note (face amount, $210000) was exchanged solely for cash; no other rights or privileges were exchanged. The note is to be repaid on December 31, 2019. The prevailing rate of interest for a loan of this type is 10%. The present value of $210000 at 10% for three years is $157773. What amount of interest income should Ms. Price recognize in 2017 __is a mechanical force generated by a solid object moving through a fluid. A client has a BMI of 24.8, which of the following categories would they fall into? The amount of juice in a container isnormally distributed with a mean of70 ounces and a standard deviation of0.5 ounce. What is the probability that arandomly selected container has morethan 70.5 ounces of juice? The addiction many Americans have to online shopping is a struggle, and the struggle is real Sara didnt get a raise in her salary,so she ....... to find another job.a-is trying b-will try Find the perimeter of this figure. (Image down below) Which of these relations are functions?O-2 6 2 -6-5 21 15-15y11y42Ox4-24o{(-5,-7), (-2,-7), (7,17), (-5,21)}y 2 pts Question 1 Write an expression to model the phrase: Myles has $635 and is earning $120 each week as a lifeguard. (Use x as your variable) 2 pts Wuestion 2 Brian needs to paint a logo using two right triangles. The dimensions of the logo are shown below. What is the difference between the area of the large triangle and the area of the small triangle? Un cultivo de bacterias crece y se duplica cada dos horas .Cu`antas veces mas grande sera el numero de bacterias en 12 horas?El numero inicial es X. Expresa usando potencias ayuda profis es para hoy ; )